ldanny24
Thanks Received: 4
Vinny Gambini
Vinny Gambini
 
Posts: 21
Joined: February 08th, 2011
 
 
 

Q16 - Motorcoach driver: Professional drivers spend

by ldanny24 Sat Mar 05, 2011 2:58 pm

Hi, I'm having trouble understanding why A is not the correct answer. I can see on further reading that C would better capture the point at issue but couldn't you also say that A would be correct? The motorcoach driver obviously states he is against the reduction of the speed limit, and the police officer is arguing for the case that the motorcoach driver is wrong. So couldn't we reasonably say that he's for the reduction of the speed limit??
User avatar
 
ManhattanPrepLSAT1
Thanks Received: 1909
Atticus Finch
Atticus Finch
 
Posts: 2851
Joined: October 07th, 2009
 
 
 

Re: Q16 - Motorcoach driver: Professional drivers spend

by ManhattanPrepLSAT1 Mon Mar 07, 2011 3:47 pm

I can see why it'd be tempting to stretch the police officer's statements that far. But the police officer is not challenging the motorcoach driver's conclusion that "the speed limit on major highways should not be reduced." Rather, the police officer is challenging the evidence the motorcoach driver uses in support of that conclusion.

If the police officer was challenging the motorcoach driver's conclusion, you'd be correct. Notice that the police officer doesn't use opinionated claims involving words like "should." The police officer's claim is about could happen - NOT what should happen.

Does that answer your question about where answer choice (A) goes wrong?
 
Elizabeth.Naff
Thanks Received: 0
Forum Guests
 
Posts: 9
Joined: June 07th, 2012
 
 
 

Re: Q16 - Motorcoach driver: Professional drivers spend

by Elizabeth.Naff Sun Jul 29, 2012 11:49 pm

I did choose the correct answer. But I had difficulty seeing how B and C fundamentally differ.

Is it because B uses the term "will" when the police officer only said "can?"

Is that really what makes this answer different?

Thanks!!
 
soyeonjeon
Thanks Received: 2
Forum Guests
 
Posts: 67
Joined: October 25th, 2012
 
 
 

Re: Q16 - Motorcoach driver: Professional drivers spend

by soyeonjeon Wed Apr 24, 2013 9:56 am

I chose B for the answer and didnt read past B unfortunately.
But doesn't the police officer's statement lead quite naturally to the conclusion that goes against what the motor driver is saying, which thereby ?
That is, by attacking A's premise, isn't B undermining A's conclusion as well, thereby denying it?

and shouldn't we read the conclusions implied there?
Please HELP ME.
 
sumukh09
Thanks Received: 139
Atticus Finch
Atticus Finch
 
Posts: 327
Joined: June 03rd, 2012
 
 
trophy
Most Thanked
trophy
First Responder
 

Re: Q16 - Motorcoach driver: Professional drivers spend

by sumukh09 Wed Apr 24, 2013 12:50 pm

soyeonjeon Wrote:I chose B for the answer and didnt read past B unfortunately.
But doesn't the police officer's statement lead quite naturally to the conclusion that goes against what the motor driver is saying, which thereby ?
That is, by attacking A's premise, isn't B undermining A's conclusion as well, thereby denying it?

and shouldn't we read the conclusions implied there?
Please HELP ME.


The police officer says that "All drivers CAN drive..." but B suggests that professional drivers will in fact obey the speed limit when the officer in no way implies this. All the officer is saying is that people, in general, can choose to obey the speed limit so reducing the speed limit should not cause anyone to break the law.
User avatar
 
ManhattanPrepLSAT1
Thanks Received: 1909
Atticus Finch
Atticus Finch
 
Posts: 2851
Joined: October 07th, 2009
 
 
 

Re: Q16 - Motorcoach driver: Professional drivers spend

by ManhattanPrepLSAT1 Thu Apr 25, 2013 3:53 pm

Just a quick note on answer choice (B)...

Notice that in answer choice (B) we get the word "if." In answer choice (C) we get the word "cause." The difference between the two is that answer choice (B) doesn't suggest a cause for breaking the speed limit whereas the issue between the two speakers is whether reducing the speed limit will cause some people to break the law.

Another difference is that answer choice (B) says that some drivers will drive within the speed limit, whereas answer choice (C) says that some drivers will break the speed limit.

I think both speakers would agree that some drivers would drive within the speed limit, the issue is whether reducing the speed limit will cause some drivers to excede the speed limit.

Hope that helps!
 
roflcoptersoisoi
Thanks Received: 0
Atticus Finch
Atticus Finch
 
Posts: 165
Joined: April 30th, 2015
 
 
 

Re: Q16 - Motorcoach driver: Professional drivers spend

by roflcoptersoisoi Sat Jul 02, 2016 6:57 pm

Point of issue:
Motorcoach driver:
Reducing speed limit will have the undesirable effect of forcing some law abiding people who are competent drivers to break the law

Police officer:
Reducing the speed limit would not force those people to break the law.

(A) Tempting, but this is a point unique to the motor coach driver's argument. The desirability of reducing the speed limit is not discussed by the police officer.
(B) This is the opposite of a point that is unique to the motorcoach driver. That reducing the speed limit will force some professional drivers to exceed the speed limit.
(C) Bingo.
(D) This is unique to the motor coach's argument.
(E) Neither of the two makes this claim.